Ch 9

B. market.

In the context of the Capital Asset Pricing Model (CAPM) the relevant measure of risk is
A. unique risk.
B. market.
C. standard deviation of returns.
D. variance of returns.

B. 1.

The market portfolio has a beta of
A. 0.
B. 1.
C. -1.
D. 0.5.

D. 0.132%.

The risk-free rate and the expected market rate of return are 0.06% and 0.12%, respectively. According to the capital asset pricing model (CAPM), the expected rate of return on security X with a beta of 1.2 is equal to
A. 0.06%.
B. 0.144%.
C. 0.12%.
D. 0.

E. I, II, and III

Which statement is true regarding the market portfolio?
I) It includes all publicly traded financial assets.
II) It lies on the efficient frontier.
III) All securities in the market portfolio are held in proportion to their market values.
IV) It is the ta

C. The CML is also called the security market line.

Which statement is not true regarding the capital market line (CML)?
A. The CML is the line from the risk-free rate through the market portfolio.
B. The CML is the best attainable capital allocation line.
C. The CML is also called the security market line

A. the covariance between the security's return and the market return divided by the variance of the market's returns.

The market risk, beta, of a security is equal to
A. the covariance between the security's return and the market return divided by the variance of the market's returns.
B. the covariance between the security and market returns divided by the standard devia

D. the line that represents the expected return-beta relationship.

The security market line (SML) is
A. the line that describes the expected return-beta relationship for well-diversified portfolios only.
B. also called the capital allocation line.
C. the line that is tangent to the efficient frontier of all risky assets.

B. zero alphas.

According to the Capital Asset Pricing Model (CAPM), fairly priced securities have
A. positive betas.
B. zero alphas.
C. negative betas.
D. positive alphas.

C. negative alphas.

According to the Capital Asset Pricing Model (CAPM), overpriced securities have
A. positive betas.
B. zero alphas.
C. negative alphas.
D. positive alphas.

C. fairly priced.

Your personal opinion is that a security has an expected rate of return of 0.11. It has a beta of 1.5. The risk-free rate is 0.05 and the market expected rate of return is 0.09. According to the Capital Asset Pricing Model, this security is
A. underpriced

B. sell short the stock because it is overpriced.

The risk-free rate is 7%. The expected market rate of return is 15%. If you expect a stock with a beta of 1.3 to offer a rate of return of 12%, you should
A. buy the stock because it is overpriced.
B. sell short the stock because it is overpriced.
C. sell

D. 1.08.

You invest $600 in a security with a beta of 1.2 and $400 in another security with a beta of 0.90. The beta of the resulting portfolio is
A. 1.40.
B. 1.00.
C. 0.36.
D. 1.08.
E. 0.80.

B. overpriced.

Your opinion is that Boeing has an expected rate of return of 0.08. It has a beta of 0.92. The risk-free rate is 0.04 and the market expected rate of return is 0.10. According to the Capital Asset Pricing Model, this security is
A. underpriced.
B. overpri

A. 13.8%.

As a financial analyst, you are tasked with evaluating a capital budgeting project. You were instructed to use the IRR method and you need to determine an appropriate hurdle rate. The risk-free rate is 4% and the expected market rate of return is 11%. You

C. A because it offers an expected excess return of 2.2%.

Given are the following two stocks A and B:
Security A E(r)=.12 B=1.2
Security B E(r)=.14 B=1.8
If the expected market rate of return is 0.09 and the risk-free rate is 0.05, which security would be considered the better buy and why? Hint: Look into excess

D. It is the tangency point between the capital market line and the indifference curve.

Which statement is not true regarding the market portfolio?
A. It includes all publicly traded financial assets.
B. It lies on the efficient frontier.
C. All securities in the market portfolio are held in proportion to their market values.
D. It is the ta

C. unsystematic risk is negligible.

In a well-diversified portfolio
A. market risk is negligible.
B. systematic risk is negligible.
C. unsystematic risk is negligible.
D. nondiversifiable risk is negligible.

B. sell short the stock because it is overpriced.

The risk-free rate is 7%. The expected market rate of return is 15%. If you expect a stock with a beta of 1.3 to offer a rate of return of 12%, you should
A. buy the stock because it is overpriced.
B. sell short the stock because it is overpriced.
C. sell